logowanie

matematyka » forum » forum zadaniowe - uczelnie wyższe » zadanie

Analiza matematyczna, zadanie nr 957

ostatnie wiadomości  |  regulamin  |  latex

AutorZadanie / Rozwiązanie

ania16177
postów: 49
2013-01-26 16:14:33

$\lim_{n \to \infty} \sqrt[n]{(\frac{3}{5})^{n} +(\frac{2}{3})^{n} +(\frac{1}{4})^{n}}$

Wiadomość była modyfikowana 2013-01-26 22:02:33 przez ania16177

tumor
postów: 8070
2013-01-27 02:04:02

$\frac{2}{3}= \sqrt[n]{(\frac{2}{3})^n}\le\sqrt[n]{(\frac{3}{5})^n+(\frac{2}{3})^n+(\frac{1}{4})^n}\le \sqrt[n]{3*(\frac{2}{3})^n}=\sqrt[n]{3}*\frac{2}{3}$

$\lim_{n \to \infty}\sqrt[n]{3}*\frac{2}{3}=\frac{2}{3}$


zatem z twierdzenia o trzech ciągach także

$\lim_{n \to \infty}\sqrt[n]{(\frac{3}{5})^n+(\frac{2}{3})^n+(\frac{1}{4})^n}=\frac{2}{3}$

strony: 1

Prawo do pisania przysługuje tylko zalogowanym użytkownikom. Zaloguj się lub zarejestruj





© 2019 Mariusz Śliwiński      o serwisie | kontakt   drukuj